Quantcast
  • Register
PhysicsOverflow is a next-generation academic platform for physicists and astronomers, including a community peer review system and a postgraduate-level discussion forum analogous to MathOverflow.

Welcome to PhysicsOverflow! PhysicsOverflow is an open platform for community peer review and graduate-level Physics discussion.

Please help promote PhysicsOverflow ads elsewhere if you like it.

News

PO is now at the Physics Department of Bielefeld University!

New printer friendly PO pages!

Migration to Bielefeld University was successful!

Please vote for this year's PhysicsOverflow ads!

Please do help out in categorising submissions. Submit a paper to PhysicsOverflow!

... see more

Tools for paper authors

Submit paper
Claim Paper Authorship

Tools for SE users

Search User
Reclaim SE Account
Request Account Merger
Nativise imported posts
Claim post (deleted users)
Import SE post

Users whose questions have been imported from Physics Stack Exchange, Theoretical Physics Stack Exchange, or any other Stack Exchange site are kindly requested to reclaim their account and not to register as a new user.

Public \(\beta\) tools

Report a bug with a feature
Request a new functionality
404 page design
Send feedback

Attributions

(propose a free ad)

Site Statistics

205 submissions , 163 unreviewed
5,047 questions , 2,200 unanswered
5,345 answers , 22,709 comments
1,470 users with positive rep
816 active unimported users
More ...

  Regulators and uniqueness

+ 5 like - 0 dislike
1996 views

Does the regularization of a divergent infinite sum yield a unique value?

I.e. do different regularization schemes acting on the same infinite sum produce the same exact value independent of the regulator?

What, exactly, do these values mean? Or what are they? My understanding is that they are not "convergent" values.


This post imported from StackExchange Mathematics at 2016-07-10 19:41 (UTC), posted by SE-user user122066

asked Jul 10, 2016 in Mathematics by user122066 (45 points) [ revision history ]
retagged Jul 10, 2016
Dangit, you beat me to it! math.stackexchange.com/questions/1854642/…

This post imported from StackExchange Mathematics at 2016-07-10 19:41 (UTC), posted by SE-user tparker
Relevant, as source/motivation of this question: physics.stackexchange.com/a/267243

This post imported from StackExchange Mathematics at 2016-07-10 19:41 (UTC), posted by SE-user Clement C.

1 Answer

+ 3 like - 0 dislike

Different regularizations may lead to different regularized values. For instance $$ \lim_{\lambda\to 0}\sum_{n\geq 0}n e^{-\lambda n} = +\infty $$ while the zeta regularization of $\sum_{n\geq 1} n $ gives the (in)famous value $\zeta(-1)=-\frac{1}{12}$.

If we take an hybrid between smoothed sums and the zeta regulatization we have: $$\sum_{n\geq 1}'' n = \sum_{N\geq 1}'\frac{N+1}{2} = \frac{\zeta(0)+\zeta(-1)}{2}=-\frac{7}{24}.$$

We also have a class of regularizations that depends on a positive parameter $\delta$: the Bochner-Riesz mean. There isn't a single regularization: a regularization is just a (somewhat arbitrary) way to extend the concept of convergence. About integrals, the Cauchy principal value can be interpreted as the Fourier transform of a distribution. About series, we may say that $$ \sum_{n\geq 1}' a_n = L$$ à-la-Cesàro if $$\lim_{N\to +\infty}\frac{A_1+\ldots+A_N}{N}=L,$$ i.e. if the sequence of partial sums is converging on average. A convergent series is also a Cesàro-convergent series, but with such an extension $$ {\sum_{n\geq 0}}'(-1)^n = \frac{1}{2}=\lim_{\lambda\to 0}\sum_{n\geq 0}(-1)^n e^{-\lambda n}$$ where $\sum_{n\geq 0}(-1)^n$ is not convergent in the usual sense.

This post imported from StackExchange Mathematics at 2016-07-10 19:41 (UTC), posted by SE-user Jack D'Aurizio
answered Jul 10, 2016 by Jack D'Aurizio (30 points) [ no revision ]
I would say that your example is an instance where one regulator "works" and another "fails to work." But are there any examples where two different regulators give different, finite values?

This post imported from StackExchange Mathematics at 2016-07-10 19:41 (UTC), posted by SE-user tparker
Could you explain your notation - what do the single and double primes on the sums mean? How did you go from summing over $n$ to summing over $(n+1)/2$?

This post imported from StackExchange Mathematics at 2016-07-10 19:41 (UTC), posted by SE-user tparker
@tparker: at first we perform a Cesàro regularization: the main term ($n$) is replaced by the averaged term $\frac{1}{N}\sum_{n=1}^{N}n = \frac{N+1}{2}$. A zeta regularization follows.

This post imported from StackExchange Mathematics at 2016-07-10 19:41 (UTC), posted by SE-user Jack D'Aurizio
I see. I would suggest getting rid of the primes on the $\Sigma$ symbols because they don't seem to mean anything and I find them extremely confusing.

This post imported from StackExchange Mathematics at 2016-07-10 19:41 (UTC), posted by SE-user tparker
@tparker: without primes the chain of equality is simply wrong, so I prefer to leave the primes there.

This post imported from StackExchange Mathematics at 2016-07-10 19:41 (UTC), posted by SE-user Jack D'Aurizio
What do the primes actually mean? What is the general distinction between $\Sigma'$ and $\Sigma''$?

This post imported from StackExchange Mathematics at 2016-07-10 19:41 (UTC), posted by SE-user tparker
$''$ means Cesàro+Zeta regularization, a single $'$ means Zeta regularization or Cesàro regularization, depending on the context.

This post imported from StackExchange Mathematics at 2016-07-10 19:41 (UTC), posted by SE-user Jack D'Aurizio
And there is no "general dinstinction" because we are just inventing regularizations for proving a non-uniqueness theorem.

This post imported from StackExchange Mathematics at 2016-07-10 19:41 (UTC), posted by SE-user Jack D'Aurizio
I'm not convinced that this is really in the spirit of Cesaro regularization, which regularizes a series, not individual terms in a series. Can you come up with a more straightforward regularization where you find a series of analytic functions $f_n(z)$ such that $f_n(0) = n$ and whose sum converges for some open set of $z$, then analytically continue the sum to $z = 0$? I checked your original attempt in the comment you deleted, and I think you made a mistake and it doesn't converge.

This post imported from StackExchange Mathematics at 2016-07-10 19:41 (UTC), posted by SE-user tparker
@tparker: all right, let me rephrase: we consider an hybrid between a particular smoothing (the main term is replaced by the average term) and the zeta regularization. Hope it is ok now. Out of curiosity, what is the purpose of this almost-non-math?

This post imported from StackExchange Mathematics at 2016-07-10 19:41 (UTC), posted by SE-user Jack D'Aurizio
In physics, we often use the analytic-continuation regularization prescription that I just described to assign values to divergent series, and I'm curious if the prescription always yields a unique answer. Unfortunately, the question appears to remain open.

This post imported from StackExchange Mathematics at 2016-07-10 19:41 (UTC), posted by SE-user tparker
@tparker: since the zeta regularition is not the only regularization, I think the question is pretty trivial, but I was interested to the problem in itself: if a series/integral is not converging in the usual sense, what is the physical meaning of taking any regularization of it? A regularization is a kind of arbitrary choice, like a choice of weights. The zeta regularization is perhaps God's weight choice? :D

This post imported from StackExchange Mathematics at 2016-07-10 19:41 (UTC), posted by SE-user Jack D'Aurizio
That's my question as well. Physicists never use the hybrid smoothing-and-regularization process you described, but they do often use the scheme I described. But if there exists a sum such that the scheme results in two different finite answers, that seems to pose serious questions about the validity of the physical theory, as you point out. I have revised my question math.stackexchange.com/questions/1854642/… to focus on this narrower question, and I believe it is no longer a duplicate of this one.

This post imported from StackExchange Mathematics at 2016-07-10 19:41 (UTC), posted by SE-user tparker

Your answer

Please use answers only to (at least partly) answer questions. To comment, discuss, or ask for clarification, leave a comment instead.
To mask links under text, please type your text, highlight it, and click the "link" button. You can then enter your link URL.
Please consult the FAQ for as to how to format your post.
This is the answer box; if you want to write a comment instead, please use the 'add comment' button.
Live preview (may slow down editor)   Preview
Your name to display (optional):
Privacy: Your email address will only be used for sending these notifications.
Anti-spam verification:
If you are a human please identify the position of the character covered by the symbol $\varnothing$ in the following word:
p$\hbar$ysics$\varnothing$verflow
Then drag the red bullet below over the corresponding character of our banner. When you drop it there, the bullet changes to green (on slow internet connections after a few seconds).
Please complete the anti-spam verification




user contributions licensed under cc by-sa 3.0 with attribution required

Your rights
...